Международная олимпиада 2022, Осло, Норвегия, 2022 год


Найдите все тройки $(a,b,p)$ целых положительных чисел, такие что число $p$ простое и $a^p=b!+p.$
посмотреть в олимпиаде

Комментарий/решение:

пред. Правка 4   4
2023-03-01 13:50:29.0 #

$a^p-p \equiv a,0 \pmod {p} = b! \equiv a,0 \pmod {p}$ по МТФ

пусть $a^p-p \equiv a \pmod {p} = b! \equiv a \pmod {p}$ тогда $b<p$

если $a>b$ то $a^p-p>b!$ поэтому $b>a$ тогда $b!\equiv 0 \pmod {a}=a^p\equiv 0 \pmod {a} -p\equiv p\pmod {a}$ но $p>a$ значит $p \ne\equiv 0 \pmod {a}$

пусть $a^p\equiv 0 \pmod {p} \Rightarrow b!\equiv 0 \pmod {p}$

разбирая $p=2,p=3 $ выходят такие ответы $(a,b,p)=(2,2,2),(3,4,3)$

пусть $p>3$ тогда $b>p>3$

Докажем что $a=p$ тогда $a=px$ где $x\geq 3$ $p(p^{p-1}x^p-1)=b!$ но тогда $(p^{p-1}x^p-1)\equiv 0 \pmod {2,3,4,5....p-1,p+1....,p+k}$ но если т.к. $x\geq 3$ то $(p^{p-1}x^p-1)\equiv 0 \pmod {2,3,4,5....p-1,p+1....,p+k}$ не делится на $x$ а должно т.к. $b>x$ потому что если $a>b $ то $px>2p>b>p$ где $x\geq 3$ но тогда $p^px^p-p>b!$т.к.$\Rightarrow \dfrac {2xp}{3}>b$,$\dfrac {(2xp)^p}{(3)^p}>b^p$,$x^pp^p-p>\dfrac {(2xp)^p}{(3)^p}>b^p$ так что $b>a$ $\Rightarrow a=p$ теперь надо доказать что при $p>3$ решений нету

помогите добить задачу я только до этого смог дойти

(3 дня решал только до этого дошел)

добивка $p>5$$p^p-p\equiv 0 \pmod{2^{p-1}}$ или меньше больше не может но заметим что при $b>5$ у него $b!=2^kx$,$k\geq 2^p$$\Rightarrow b\leq4$

пред. Правка 2   0
2023-03-08 23:49:20.0 #

  5
2023-12-04 00:47:47.0 #

Предположим, $p>10.$

$\textit{(i)}$

Если $b\geq 2p$, то $v_p(b!)\geq 2$, следовательно, $v_p(b!+p)=1$, следовательно, $v_p(a^p)=1.$

Это означает, что $p\mid a^p$, откуда следует, что $p\mid a$, поэтому $v_p(a^p)=pv_p(a)=p.$

Следовательно, $p=1$, но это абсурд.

$\textit{(ii)}$

Если $b<p.$ В этом случае $b!<b^p\Rightarrow b!+b<b^p+p\Rightarrow a^p<b^p+p.$

Поскольку идеальные степени $p$ должны находиться на расстоянии не менее $p+1$, мы имеем, что $a^p<b^p\Rightarrow a<b.$

Это означает, что $a\mid b!$, следовательно, $a\mid p$ и либо $a=1$, либо $a=p.$

$a=1$ не работает. Если $a=p$, то $p^p=b!+p$, поэтому аргумент размера сразу же приводит к противоречию.

$\textit{(iii)}$ Если $p\leq b<2p.$

Если существует простое число $q$ такое, что $q\neq p$ и $q\mid a$, то $q>b\geq p.$

Более того, $a^p\geq q^pp^p>p^{2p}>(2p-1)!+p>b!+p.$

Следовательно, такого $q$ не может существовать. Следовательно, $a=p^\alpha.$ Проверив этот аргумент размера, вы увидите, что у нас должно быть $\alpha=1.$ Таким образом, $a=p.$

Используя LTE, мы имеем $v_2(p^{p-1}-1)=v_2(p-1)+v_2(p+1)+v_2(p-1)-1=v_2(b!).$

Используя эту формулу Лежандра или как она там называется, мы получаем $v_2(b!)=b-s_2(b).$

Поскольку $p\leq b<2p$, получаем $v_2(b!)=b-s_2(b)\geq p-\log_2(2p).$

С другой стороны, $2\log_2(p-1)+\log(p+1)-1\geq 2v_2(p-1)+v_2(p+1)-1.$

Объединив все это, мы получаем $$2\log_2(p-1)+\log_2(p+1)-1\geq p-\log_2(2p)$$Но это неверно для $p>10.$ Таким образом, мы получим желаемое противоречие.

Теперь мы проверяем $p=2,3,5,7$, где $b$ находится в соответствующем диапазоне.